kkk Flashcards

mcq

1
Q
  1. Which of the following regurgitant fractions would be considered severe aortic valve regurgitation?
    A. 30%
    B. 40%
    C. 50% D. 70%
A
  1. Which of the following regurgitant fractions would be considered severe aortic valve regurgitation?
    A. 30%
    B. 40%
    C. 50% (answer) D. 70%
How well did you know this?
1
Not at all
2
3
4
5
Perfectly
2
Q
  1. A 23-year-old has a mild-diastolic rumble and sharp early diastolic sound. What is the likely explanation?
    A. Mitral stenosis B. Aortic stenosis
    C. Tricuspid regurgitation
    D. Pulmonary hypertension
A
  1. A 23-year-old has a mild-diastolic rumble and sharp early diastolic sound. What is the likely explanation?
    A. Mitral stenosis (answer) B. Aortic stenosis
    C. Tricuspid regurgitation
    D. Pulmonary hypertension
How well did you know this?
1
Not at all
2
3
4
5
Perfectly
3
Q
  1. A 56-year-old man comes to the emergency department with acute heart failure. What is the most appropriate initial management?
    A. Intravenous nitroglycerin B. Oral furosemide
    C. Subcutaneous heparin
    D. Immediate thrombolysis
  2. A 63-year-old man presents with exertional dyspnea, a heart murmur, and a calcified aortic valve. The mean aortic gradient is 52 mmHg, and valve area is 0.8 cm2. What is the next best step?
    A. Balloon valvuloplasty
    B. Medical management
    C. Aortic valve replacement operation after coronary angiogram D. Lifestyle modification
A
  1. A 56-year-old man comes to the emergency department with acute heart failure. What is the most appropriate initial management?
    A. Intravenous nitroglycerin (answer) B. Oral furosemide
    C. Subcutaneous heparin
    D. Immediate thrombolysis
  2. A 63-year-old man presents with exertional dyspnea, a heart murmur, and a calcified aortic valve. The mean aortic gradient is 52 mmHg, and valve area is 0.8 cm2. What is the next best step?
    A. Balloon valvuloplasty
    B. Medical management
    C. Aortic valve replacement operation after coronary angiogram (answer) D. Lifestyle modification
How well did you know this?
1
Not at all
2
3
4
5
Perfectly
4
Q
  1. Which of the following statements about angina pectoris is true?
    A. It is characterized by sharp, stabbing pain.
    B. Pain is relieved by sitting upright.
    C. It is typically described as burning, heavy, or squeezing pain relieved by nitroglycerin.
    D. It is often caused by valvular heart disease.
  2. Which drugs can cause lower extremity peripheral edema?
    A. ACE inhibitors
    B. Calcium channel blockers C. Beta blockers
    D. Diuretics
A
  1. Which of the following statements about angina pectoris is true?
    A. It is characterized by sharp, stabbing pain.
    B. Pain is relieved by sitting upright.
    C. It is typically described as burning, heavy, or squeezing pain relieved by (answer) nitroglycerin.
    D. It is often caused by valvular heart disease.
  2. Which drugs can cause lower extremity peripheral edema?
    A. ACE inhibitors
    B. Calcium channel blockers (answer) C. Beta blockers
    D. Diuretics
How well did you know this?
1
Not at all
2
3
4
5
Perfectly
5
Q
  1. Which of the following best describes the sound of mitral regurgitation?
    A. Systolic ejection murmur heard in the apical position, radiating to the axilla
    B. Continuous murmur heard over the sternal border
    C. Diastolic murmur with a loud S1
    D. Crescendo-decrescendo systolic murmur
  2. Which of the following statements about syncope is correct?
    A. It is always due to cardiac conditions.
    B. Neurally mediated syncope includes vasovagal, situational, and carotid sinus hypersensitivity.
    C. Syncope never occurs in young, healthy individuals.
    D. Syncope is not associated with bradycardia.
A
  1. Which of the following best describes the sound of mitral regurgitation?
    A. Systolic ejection murmur heard in the apical position, radiating to the axilla (answer) B. Continuous murmur heard over the sternal border
    C. Diastolic murmur with a loud S1
    D. Crescendo-decrescendo systolic murmur
  2. Which of the following statements about syncope is correct?
    A. It is always due to cardiac conditions.
    B. Neurally mediated syncope includes vasovagal, situational, and carotid sinus hypersensitivity. (answer)
    C. Syncope never occurs in young, healthy individuals.
    D. Syncope is not associated with bradycardia.
How well did you know this?
1
Not at all
2
3
4
5
Perfectly
6
Q
  1. A murmur is heard at the fourth left intercostal space along the left sternal border. Which valve is likely involved?
    A. Aortic valve
    B. Mitral valve
    C. Tricuspid valve D. Pulmonary valve
  2. Which type of heart sound is expected in a patient with mitral stenosis?
    A. A systolic ejection click
    B. A high-pitched diastolic murmur best heard at the cardiac apex C. A low-pitched diastolic murmur best heard at the apex D. A continuous murmur over the sternal border
A
  1. A murmur is heard at the fourth left intercostal space along the left sternal border. Which valve is likely involved?
    A. Aortic valve
    B. Mitral valve
    C. Tricuspid valve (answer) D. Pulmonary valve
  2. Which type of heart sound is expected in a patient with mitral stenosis?
    A. A systolic ejection click
    B. A high-pitched diastolic murmur best heard at the cardiac apex C. A low-pitched diastolic murmur best heard at the apex (answer) D. A continuous murmur over the sternal border
How well did you know this?
1
Not at all
2
3
4
5
Perfectly
7
Q
  1. Which of the following describes splinter hemorrhages?
    A. Horizontal lines under the nails due to trauma
    B. Tiny vertical blood clots under the nails, often associated with infective endocarditis
    C. Nail discoloration due to liver disease
    D. Longitudinal ridges due to vitamin deficiency
  2. Which clinical sign is associated with lower extremity peripheral venous disease?
    A. Decreased pulses
    B. Skin ulcers and edema C. Pain relieved by walking
    D. Cold, cyanotic toes
A
  1. Which of the following describes splinter hemorrhages?
    A. Horizontal lines under the nails due to trauma
    B. Tiny vertical blood clots under the nails, often associated with infective endocarditis (answer)
    C. Nail discoloration due to liver disease
    D. Longitudinal ridges due to vitamin deficiency
  2. Which clinical sign is associated with lower extremity peripheral venous disease?
    A. Decreased pulses
    B. Skin ulcers and edema (answer) C. Pain relieved by walking
    D. Cold, cyanotic toes
How well did you know this?
1
Not at all
2
3
4
5
Perfectly
8
Q
  1. Which of the following is considered asymptomatic organ damage in hypertension?
    A. Pulse pressure >60 mmHg B. Resting heart rate >90 bpm
    C. Systolic BP >180 mmHg
    D. Presence of retinal hemorrhages
  2. Which of the following statements about pulsus paradoxus is correct?
    A. It is an inspiratory increase in BP >10 mmHg.
    B. It is an abnormally large inspiratory decline (>10 mmHg) in systemic BP. C. It is a normal physiological response to deep breathing.
    D. It is primarily associated with mitral stenosis.
A
  1. Which of the following is considered asymptomatic organ damage in hypertension?
    A. Pulse pressure >60 mmHg (answer) B. Resting heart rate >90 bpm
    C. Systolic BP >180 mmHg
    D. Presence of retinal hemorrhages
  2. Which of the following statements about pulsus paradoxus is correct?
    A. It is an inspiratory increase in BP >10 mmHg.
    B. It is an abnormally large inspiratory decline (>10 mmHg) in systemic BP. (answer) C. It is a normal physiological response to deep breathing.
    D. It is primarily associated with mitral stenosis.
How well did you know this?
1
Not at all
2
3
4
5
Perfectly
9
Q
  1. Which of the following statements about cyanosis is correct?
    A. Central cyanosis is caused by vasoconstriction in extremities.
    B. Central cyanosis is caused by diseases of the heart or lungs, or abnormal hemoglobin. C. Peripheral cyanosis is seen in severe anemia.
    D. Cyanosis is always indicative of hypoxemia.
  2. Which of the following is a precipitating cause of high-output cardiac failure?
    A. Hypothyroidism
    B. Thyrotoxicosis C. Pulmonary hypertension D. Aortic stenosis
A
  1. Which of the following statements about cyanosis is correct?
    A. Central cyanosis is caused by vasoconstriction in extremities. (answer)
    B. Central cyanosis is caused by diseases of the heart or lungs, or abnormal hemoglobin. C. Peripheral cyanosis is seen in severe anemia.
    D. Cyanosis is always indicative of hypoxemia.
  2. Which of the following is a precipitating cause of high-output cardiac failure?
    A. Hypothyroidism
    B. Thyrotoxicosis (answer) C. Pulmonary hypertension D. Aortic stenosis
How well did you know this?
1
Not at all
2
3
4
5
Perfectly
10
Q
  1. Arterial thrombosis after plaque rupture is initiated by which factor?
    A. Collagen
    B. Platelet-activating factor
    C. Tissue factor
    D. Endothelin
  2. Which one of the following statements about heart failure is false?
    A. Most patients have heart failure with preserved LV systolic function.
    B. Heart failure can result from diastolic dysfunction.
    C. Pulmonary edema is a common complication.
    D. Increased preload is a compensatory mechanism in heart failure.
A
  1. Arterial thrombosis after plaque rupture is initiated by which factor?
    A. Collagen
    B. Platelet-activating factor C. Tissue factor (answer) D. Endothelin
  2. Which one of the following statements about heart failure is false?
    A. Most patients have heart failure with preserved LV systolic function. (answer)
    B. Heart failure can result from diastolic dysfunction.
    C. Pulmonary edema is a common complication.
    D. Increased preload is a compensatory mechanism in heart failure.
How well did you know this?
1
Not at all
2
3
4
5
Perfectly
11
Q
  1. Which myocardial necrosis marker raises at the very last after an ST elevation MI?
    A. Troponin I
    B. Lactate dehydrogenase (LDH) C. Creatine kinase (CK)
    D. Myoglobin
  2. Which one of the following markers rises earliest after an ST elevation MI?
    A. Troponin T
    B. Lactate dehydrogenase (LDH) C. Creatine kinase (CK)
    D. Myoglobin
A
  1. Which myocardial necrosis marker raises at the very last after an ST elevation MI?
    A. Troponin I
    B. Lactate dehydrogenase (LDH) (answer) C. Creatine kinase (CK)
    D. Myoglobin
  2. Which one of the following markers rises earliest after an ST elevation MI?
    A. Troponin T
    B. Lactate dehydrogenase (LDH) C. Creatine kinase (CK)
    D. Myoglobin (answer)
How well did you know this?
1
Not at all
2
3
4
5
Perfectly
12
Q
  1. A 55-year-old man undergoes an exercise treadmill test. His Duke treadmill score is -6. What does this indicate?
    A. Low risk of coronary artery disease
    B. Moderate risk of coronary artery disease
    C. High risk of coronary artery disease D. No clinical significance
  2. Which of the following is a risk factor for sudden death in hypertrophic cardiomyopathy?
    A. Asymptomatic bradycardia
    B. Syncope during exercise C. Resting hypertension
    D. None of the above
A
  1. A 55-year-old man undergoes an exercise treadmill test. His Duke treadmill score is -6. What does this indicate?
    A. Low risk of coronary artery disease
    B. Moderate risk of coronary artery disease
    C. High risk of coronary artery disease (answer) D. No clinical significance
  2. Which of the following is a risk factor for sudden death in hypertrophic cardiomyopathy?
    A. Asymptomatic bradycardia
    B. Syncope during exercise (answer) C. Resting hypertension
    D. None of the above
How well did you know this?
1
Not at all
2
3
4
5
Perfectly
13
Q
  1. The absolute indications for terminating an exercise test include all of the following except:
    A. New chest pain
    B. A fall in systolic BP >10 mmHg
    C. Sustained ventricular tachycardia
    D. Increased exercise capacity
  2. A 78-year-old man presents with progressive dyspnea (NYHA Class III). In addition to an echocardiogram, what is the next appropriate test to assess heart failure?
    A. Troponin I levels
    B. Brain natriuretic peptide (BNP) measurement C. Chest X-ray
    D. Right heart catheterization
A
  1. The absolute indications for terminating an exercise test include all of the following except:
    A. New chest pain
    B. A fall in systolic BP >10 mmHg
    C. Sustained ventricular tachycardia
    D. Increased exercise capacity (answer)
  2. A 78-year-old man presents with progressive dyspnea (NYHA Class III). In addition to an echocardiogram, what is the next appropriate test to assess heart failure?
    A. Troponin I levels
    B. Brain natriuretic peptide (BNP) measurement (answer) C. Chest X-ray
    D. Right heart catheterization
How well did you know this?
1
Not at all
2
3
4
5
Perfectly
14
Q
  1. Sudden cardiac death in patients >35 years old is most commonly associated with:
    A. Cardiomyopathy
    B. Coronary artery disease C. Aortic dissection
    D. Hypertensive emergencies
  2. Which form of congenital heart disease is commonly associated with Wolff-Parkinson- White syndrome?
    A. Tetralogy of Fallot
    B. Ebstein anomaly C. Patent ductus arteriosus D. Coarctation of the aorta
A
  1. Sudden cardiac death in patients >35 years old is most commonly associated with:
    A. Cardiomyopathy
    B. Coronary artery disease (answer) C. Aortic dissection
    D. Hypertensive emergencies
  2. Which form of congenital heart disease is commonly associated with Wolff-Parkinson- White syndrome?
    A. Tetralogy of Fallot
    B. Ebstein anomaly (answer) C. Patent ductus arteriosus D. Coarctation of the aorta
How well did you know this?
1
Not at all
2
3
4
5
Perfectly
15
Q
  1. Which of the following statements about coronary arteries is true?
    A. A dominant left coronary artery supplies the posterior descending artery.
    B. A dominant right coronary artery gives rise to the posterior descending artery. C. The left coronary artery always supplies the AV node.
    D. Coronary dominance does not affect perfusion territories.
  2. All of the following are high-risk features of the TIMI risk score except:
    A. Tachycardia
    B. Elevated cardiac biomarkers
    C. Age >65 years
    D. Recent use of aspirin
A
  1. Which of the following statements about coronary arteries is true?
    A. A dominant left coronary artery supplies the posterior descending artery.
    B. A dominant right coronary artery gives rise to the posterior descending artery. (answer) C. The left coronary artery always supplies the AV node.
    D. Coronary dominance does not affect perfusion territories.
  2. All of the following are high-risk features of the TIMI risk score except:
    A. Tachycardia (answer)
    B. Elevated cardiac biomarkers
    C. Age >65 years
    D. Recent use of aspirin
How well did you know this?
1
Not at all
2
3
4
5
Perfectly
16
Q
  1. Which of the following is not included in the differential diagnosis for ECG ST elevations?
    A. Pericarditis
    B. Left ventricular hypertrophy C. Coronary vasospasm
    D. Aortic stenosis
  2. Which of the following is NOT a common cause of continuous murmurs?
    A. Patent ductus arteriosus
    B. Arteriovenous fistula
    C. Aortic stenosis and regurgitation D. Coarctation of the aorta
A
  1. Which of the following is not included in the differential diagnosis for ECG ST elevations?
    A. Pericarditis
    B. Left ventricular hypertrophy (answer) C. Coronary vasospasm
    D. Aortic stenosis
  2. Which of the following is NOT a common cause of continuous murmurs?
    A. Patent ductus arteriosus
    B. Arteriovenous fistula
    C. Aortic stenosis and regurgitation (answer) D. Coarctation of the aorta
How well did you know this?
1
Not at all
2
3
4
5
Perfectly
17
Q
  1. A 20-year-old woman has an early diastolic murmur best heard at the left sternal border during expiration. What is the most appropriate next step in management?
    A. No intervention; reassure the patient B. Echocardiography
    C. Chest X-ray
    D. Blood cultures
  2. Which marker of myocardial necrosis is most specific for myocardial infarction?
    A. Troponin I B. Myoglobin
    C. Lactate dehydrogenase (LDH)
    D. Creatine kinase (CK-MB)
A
  1. A 20-year-old woman has an early diastolic murmur best heard at the left sternal border during expiration. What is the most appropriate next step in management?
    A. No intervention; reassure the patient B. Echocardiography (answer)
    C. Chest X-ray
    D. Blood cultures
  2. Which marker of myocardial necrosis is most specific for myocardial infarction?
    A. Troponin I (answer) B. Myoglobin
    C. Lactate dehydrogenase (LDH)
    D. Creatine kinase (CK-MB)
18
Q
  1. Which of the following drugs is least likely to slow conduction through the AV node?
    A. Amiodarone
    B. Adenosine
    C. Lidocaine D. Verapamil
  2. Which of the following ECG findings is LEAST likely associated with acute pericarditis?
    A. Diffuse ST elevation B. PR depression
    C. Q waves
    D. Low voltage QRS
A
  1. Which of the following drugs is least likely to slow conduction through the AV node?
    A. Amiodarone
    B. Adenosine
    C. Lidocaine (answer) D. Verapamil
  2. Which of the following ECG findings is LEAST likely associated with acute pericarditis?
    A. Diffuse ST elevation B. PR depression
    C. Q waves (answer)
    D. Low voltage QRS
19
Q
  1. In a patient with Wolff-Parkinson-White syndrome, which of the following arrhythmias is most concerning?
    A. Atrial fibrillation B. Ventricular tachycardia
    C. Sinus bradycardia
    D. Multifocal atrial tachycardia
  2. Which of the following conditions is most likely to cause a widened pulse pressure?
    A. Aortic regurgitation B. Pulmonary stenosis
    C. Mitral stenosis
    D. Hypertrophic cardiomyopathy
A
  1. In a patient with Wolff-Parkinson-White syndrome, which of the following arrhythmias is most concerning?
    A. Atrial fibrillation (answer) B. Ventricular tachycardia
    C. Sinus bradycardia
    D. Multifocal atrial tachycardia
  2. Which of the following conditions is most likely to cause a widened pulse pressure?
    A. Aortic regurgitation (answer) B. Pulmonary stenosis
    C. Mitral stenosis
    D. Hypertrophic cardiomyopathy
20
Q
  1. In which of the following disorders is the development of pulmonary hypertension primarily due to increased pulmonary blood flow?
    A. Eisenmenger syndrome
    B. Ostium primum atrial septal defect
    C. Chronic obstructive pulmonary disease (COPD) D. Left ventricular failure
  2. A 32-year-old woman has dyspnea on exertion, an opening snap, and a mid-diastolic rumble at the apex. What is the most likely diagnosis?
    A. Mitral stenosis B. Aortic regurgitation
    C. Tricuspid regurgitation
    D. Pulmonary hypertension
A
  1. In which of the following disorders is the development of pulmonary hypertension primarily due to increased pulmonary blood flow?
    A. Eisenmenger syndrome
    B. Ostium primum atrial septal defect (answer)
    C. Chronic obstructive pulmonary disease (COPD) D. Left ventricular failure
  2. A 32-year-old woman has dyspnea on exertion, an opening snap, and a mid-diastolic rumble at the apex. What is the most likely diagnosis?
    A. Mitral stenosis (answer) B. Aortic regurgitation
    C. Tricuspid regurgitation
    D. Pulmonary hypertension
21
Q
  1. Which of the following therapies is NOT a class I recommendation for patients with unstable coronary syndromes?
    A. Dual antiplatelet therapy B. Statins
    C. Nitrate therapy D. Beta-blockers
  2. Which of the following findings is most indicative of tricuspid valve regurgitation?
    A. Pansystolic murmur that increases with inspiration B. Early diastolic murmur best heard at the apex
    C. Crescendo-decrescendo murmur at the left sternal border
    D. Systolic murmur radiating to the neck
A
  1. Which of the following therapies is NOT a class I recommendation for patients with unstable coronary syndromes?
    A. Dual antiplatelet therapy B. Statins
    C. Nitrate therapy (answer) D. Beta-blockers
  2. Which of the following findings is most indicative of tricuspid valve regurgitation?
    A. Pansystolic murmur that increases with inspiration (answer) B. Early diastolic murmur best heard at the apex
    C. Crescendo-decrescendo murmur at the left sternal border
    D. Systolic murmur radiating to the neck
22
Q
  1. A 56-year-old woman with hypertension and elevated potassium levels needs treatment for hypertension. Which medication is most appropriate?
    A. Amlodipine B. Lisinopril
    C. Spironolactone
    D. Hydrochlorothiazide
  2. Which of the following is NOT a common risk factor for stroke in patients with atrial fibrillation?
    A. Hypertension
    B. Diabetes
    C. Dyslipidemia D. Age >65
A
  1. A 56-year-old woman with hypertension and elevated potassium levels needs treatment for hypertension. Which medication is most appropriate?
    A. Amlodipine (answer) B. Lisinopril
    C. Spironolactone
    D. Hydrochlorothiazide
  2. Which of the following is NOT a common risk factor for stroke in patients with atrial fibrillation?
    A. Hypertension
    B. Diabetes
    C. Dyslipidemia (answer) D. Age >65
23
Q
  1. Which of the following is NOT a high-output cause of heart failure?
    A. Severe anemia
    B. Thyrotoxicosis
    C. Chronic kidney disease D. Paget’s disease
  2. Which of the following is TRUE regarding atrial septal defects?
    A. Most commonly present with cyanosis in infancy
    B. Always result in left-to-right shunting
    C. Commonly lead to Eisenmenger syndrome if untreated D. Are always asymptomatic in adults
A
  1. Which of the following is NOT a high-output cause of heart failure?
    A. Severe anemia
    B. Thyrotoxicosis
    C. Chronic kidney disease (answer) D. Paget’s disease
  2. Which of the following is TRUE regarding atrial septal defects?
    A. Most commonly present with cyanosis in infancy
    B. Always result in left-to-right shunting
    C. Commonly lead to Eisenmenger syndrome if untreated (answer) D. Are always asymptomatic in adults
24
Q
  1. A 76-year-old woman presents with a holosystolic murmur at the 5th intercostal space, midclavicular line, with symptoms of dyspnea and leg swelling. What is the most likely cause?
    A. Mitral regurgitation B. Aortic stenosis
    C. Pulmonary regurgitation
    D. Tricuspid stenosis
  2. Which of the following findings is associated with infective endocarditis?
    A. Janeway lesions B. Xanthomas
    C. Butterfly rash
    D. Elevated anti-dsDNA antibodies
A
  1. A 76-year-old woman presents with a holosystolic murmur at the 5th intercostal space, midclavicular line, with symptoms of dyspnea and leg swelling. What is the most likely cause?
    A. Mitral regurgitation (answer) B. Aortic stenosis
    C. Pulmonary regurgitation
    D. Tricuspid stenosis
  2. Which of the following findings is associated with infective endocarditis?
    A. Janeway lesions (answer) B. Xanthomas
    C. Butterfly rash
    D. Elevated anti-dsDNA antibodies
25
Q
  1. Which of the following is the earliest ECG finding in hyperkalemia?
    A. Flattened T waves
    B. Peaked T waves C. QRS widening
    D. ST depression
  2. Which of the following cardiac conditions is most commonly associated with carcinoid syndrome?
    A. Mitral stenosis
    B. Aortic regurgitation
    C. Tricuspid stenosis D. Pulmonary regurgitation
A
  1. Which of the following is the earliest ECG finding in hyperkalemia?
    A. Flattened T waves
    B. Peaked T waves (answer) C. QRS widening
    D. ST depression
  2. Which of the following cardiac conditions is most commonly associated with carcinoid syndrome?
    A. Mitral stenosis
    B. Aortic regurgitation
    C. Tricuspid stenosis (answer) D. Pulmonary regurgitation
26
Q
  1. What is the predominant cause of sudden cardiac death in patients <35 years old?
    A. Coronary artery disease
    B. Hypertrophic cardiomyopathy C. Aortic dissection
    D. Pulmonary embolism
  2. Which of the following markers is elevated in type 5 myocardial infarction?
    A. CK-MB
    B. Troponin I
    C. Lactate dehydrogenase (LDH) D. Brain natriuretic peptide (BNP)
A
  1. What is the predominant cause of sudden cardiac death in patients <35 years old?
    A. Coronary artery disease
    B. Hypertrophic cardiomyopathy (answer) C. Aortic dissection
    D. Pulmonary embolism
  2. Which of the following markers is elevated in type 5 myocardial infarction?
    A. CK-MB (answer)
    B. Troponin I
    C. Lactate dehydrogenase (LDH) D. Brain natriuretic peptide (BNP)
27
Q
  1. Which of the following drugs is considered first-line therapy for a patient with hyperlipidemia and triglycerides >500 mg/dL?
    A. Atorvastatin
    B. Gemfibrozil C. Ezetimibe
    D. Niacin
  2. Which of the following findings on auscultation is most likely associated with aortic stenosis?
    A. Holosystolic murmur radiating to the axilla
    B. Late-peaking crescendo-decrescendo murmur at the right upper sternal border C. Continuous murmur heard best at the left sternal border
    D. Low-pitched diastolic murmur at the apex
A
  1. Which of the following drugs is considered first-line therapy for a patient with hyperlipidemia and triglycerides >500 mg/dL?
    A. Atorvastatin
    B. Gemfibrozil (answer) C. Ezetimibe
    D. Niacin
  2. Which of the following findings on auscultation is most likely associated with aortic stenosis?
    A. Holosystolic murmur radiating to the axilla
    B. Late-peaking crescendo-decrescendo murmur at the right upper sternal border (answer) C. Continuous murmur heard best at the left sternal border
    D. Low-pitched diastolic murmur at the apex
28
Q
  1. Which of the following markers is elevated in the earliest stage of infective endocarditis?
    A. C-reactive protein (CRP) B. Erythrocyte sedimentation rate (ESR)
    C. Troponin I
    D. Lactate dehydrogenase (LDH)
  2. Which of the following is the most common cause of tricuspid regurgitation?
    A. Right ventricular dilation due to left-sided heart failure B. Pulmonary embolism
    C. Endocarditis
    D. Congenital abnormalities
A
  1. Which of the following markers is elevated in the earliest stage of infective endocarditis?
    A. C-reactive protein (CRP) (answer) B. Erythrocyte sedimentation rate (ESR)
    C. Troponin I
    D. Lactate dehydrogenase (LDH)
  2. Which of the following is the most common cause of tricuspid regurgitation?
    A. Right ventricular dilation due to left-sided heart failure (answer) B. Pulmonary embolism
    C. Endocarditis
    D. Congenital abnormalities
29
Q
  1. Which of the following ECG findings is most likely seen in a patient with pulmonary embolism?
    A. ST elevation in leads V2-V4 B. S1Q3T3 pattern C. Wide QRS complexes
    D. Peaked T waves
  2. Which of the following forms of congenital heart disease is least likely to be cyanotic in infancy?
    A. Tetralogy of Fallot
    B. Transposition of the great arteries C. Atrial septal defect
    D. Truncus arteriosus
A
  1. Which of the following ECG findings is most likely seen in a patient with pulmonary embolism?
    A. ST elevation in leads V2-V4 B. S1Q3T3 pattern (answer) C. Wide QRS complexes
    D. Peaked T waves
  2. Which of the following forms of congenital heart disease is least likely to be cyanotic in infancy?
    A. Tetralogy of Fallot
    B. Transposition of the great arteries C. Atrial septal defect (answer)
    D. Truncus arteriosus
30
Q
  1. Which of the following is the definitive treatment for hypertrophic cardiomyopathy in patients with severe symptoms?
    A. Beta-blockers
    B. Septal myectomy
    C. Implantable cardioverter-defibrillator (ICD) D. Amiodarone
  2. Which of the following statements about coronary dominance is true?
    A. Right dominance is the most common pattern. B. Left dominance leads to higher rates of ischemia.
    C. Co-dominance is associated with inferior myocardial infarctions.
    D. Coronary dominance is determined by the left anterior descending artery.
A
  1. Which of the following is the definitive treatment for hypertrophic cardiomyopathy in patients with severe symptoms?
    A. Beta-blockers
    B. Septal myectomy (answer)
    C. Implantable cardioverter-defibrillator (ICD) D. Amiodarone
  2. Which of the following statements about coronary dominance is true?
    A. Right dominance is the most common pattern. (answer) B. Left dominance leads to higher rates of ischemia.
    C. Co-dominance is associated with inferior myocardial infarctions.
    D. Coronary dominance is determined by the left anterior descending artery.
31
Q
  1. Which of the following is NOT a typical finding in cardiac tamponade?
    A. Pulsus paradoxus
    B. Elevated jugular venous pressure C. Distant heart sounds
    D. Wide pulse pressure
  2. Which of the following antiarrhythmic drugs has the least effect on prolonging the QT interval?
    A. Amiodarone
    B. Sotalol
    C. Lidocaine
    D. Dofetilide
A
  1. Which of the following is NOT a typical finding in cardiac tamponade?
    A. Pulsus paradoxus
    B. Elevated jugular venous pressure C. Distant heart sounds
    D. Wide pulse pressure (answer)
  2. Which of the following antiarrhythmic drugs has the least effect on prolonging the QT interval?
    A. Amiodarone
    B. Sotalol
    C. Lidocaine (answer)
    D. Dofetilide
32
Q
  1. Which of the following markers is used to monitor the progression of heart failure?
    A. Creatine kinase
    B. Brain natriuretic peptide (BNP) C. C-reactive protein (CRP)
    D. Troponin T
  2. A 72-year-old man is diagnosed with aortic stenosis. Which of the following features suggests severe disease?
    A. Mid-peaking systolic murmur
    B. Delayed and weak carotid pulse C. Wide pulse pressure
    D. Soft S4 sound
A
  1. Which of the following markers is used to monitor the progression of heart failure?
    A. Creatine kinase
    B. Brain natriuretic peptide (BNP) (answer) C. C-reactive protein (CRP)
    D. Troponin T
  2. A 72-year-old man is diagnosed with aortic stenosis. Which of the following features suggests severe disease?
    A. Mid-peaking systolic murmur
    B. Delayed and weak carotid pulse (answer) C. Wide pulse pressure
    D. Soft S4 sound
33
Q
  1. Which of the following conditions is NOT included in the differential diagnosis of a widened mediastinum on chest X-ray?
    A. Aortic dissection
    B. Large pericardial effusion
    C. Cardiac tamponade D. Thoracic aortic aneurysm
  2. Which of the following is a characteristic feature of pulmonary hypertension?
    A. Decreased pulmonary vascular resistance
    B. Enlarged pulmonary arteries on imaging C. Cyanosis with exercise tolerance
    D. Increased cardiac output
A
  1. Which of the following conditions is NOT included in the differential diagnosis of a widened mediastinum on chest X-ray?
    A. Aortic dissection
    B. Large pericardial effusion
    C. Cardiac tamponade (answer) D. Thoracic aortic aneurysm
  2. Which of the following is a characteristic feature of pulmonary hypertension?
    A. Decreased pulmonary vascular resistance
    B. Enlarged pulmonary arteries on imaging (answer) C. Cyanosis with exercise tolerance
    D. Increased cardiac output
34
Q
  1. Which of the following murmurs increases with Valsalva maneuver?
    A. Mitral regurgitation
    B. Hypertrophic cardiomyopathy murmur C. Tricuspid regurgitation
    D. Aortic stenosis
  2. What is the most common cause of sudden cardiac death in athletes?
    A. Myocardial infarction
    B. Hypertrophic cardiomyopathy C. Coronary artery dissection
    D. Aortic rupture
A
  1. Which of the following murmurs increases with Valsalva maneuver?
    A. Mitral regurgitation
    B. Hypertrophic cardiomyopathy murmur (answer) C. Tricuspid regurgitation
    D. Aortic stenosis
  2. What is the most common cause of sudden cardiac death in athletes?
    A. Myocardial infarction
    B. Hypertrophic cardiomyopathy (answer) C. Coronary artery dissection
    D. Aortic rupture
35
Q
  1. Which of the following drugs is most appropriate for managing hypertension in a patient with chronic kidney disease?
    A. Amlodipine
    B. Lisinopril C. Metoprolol
    D. Hydrochlorothiazide
  2. Which of the following conditions is associated with a paradoxical embolism?
    A. Patent ductus arteriosus
    B. Atrial septal defect with right-to-left shunting C. Eisenmenger syndrome with left-to-right shunting
    D. Hypertrophic cardiomyopathy
A
  1. Which of the following drugs is most appropriate for managing hypertension in a patient with chronic kidney disease?
    A. Amlodipine
    B. Lisinopril (answer) C. Metoprolol
    D. Hydrochlorothiazide
  2. Which of the following conditions is associated with a paradoxical embolism?
    A. Patent ductus arteriosus
    B. Atrial septal defect with right-to-left shunting (answer) C. Eisenmenger syndrome with left-to-right shunting
    D. Hypertrophic cardiomyopathy
36
Q
  1. Which of the following drugs is most appropriate for managing hypertension in a patient with chronic kidney disease?
    A. Amlodipine
    B. Lisinopril C. Metoprolol
    D. Hydrochlorothiazide
  2. Which of the following conditions is associated with a paradoxical embolism?
    A. Patent ductus arteriosus
    B. Atrial septal defect with right-to-left shunting C. Eisenmenger syndrome with left-to-right shunting
    D. Hypertrophic cardiomyopathy
A
  1. Which of the following drugs is most appropriate for managing hypertension in a patient with chronic kidney disease?
    A. Amlodipine
    B. Lisinopril (answer) C. Metoprolol
    D. Hydrochlorothiazide
  2. Which of the following conditions is associated with a paradoxical embolism?
    A. Patent ductus arteriosus
    B. Atrial septal defect with right-to-left shunting (answer) C. Eisenmenger syndrome with left-to-right shunting
    D. Hypertrophic cardiomyopathy
37
Q
  1. Which of the following regurgitant fractions would be considered severe aortic valve regurgitation?

    A) 30%

    B) 40%

    C) 50% D) 60%

  2. A 23-year-old has a mild-diastolic rumble and sharp early diastolic sound. What is the likely explanation?

    A) Aortic stenosis

    B) Tricuspid regurgitation

    C) Mitral stenosis 
 D) Pulmonary hypertension

A
  1. Which of the following regurgitant fractions would be considered severe aortic valve regurgitation?

    A) 30%

    B) 40%

    C) 50% (answer)
 D) 60%

  2. A 23-year-old has a mild-diastolic rumble and sharp early diastolic sound. What is the likely explanation?

    A) Aortic stenosis

    B) Tricuspid regurgitation

    C) Mitral stenosis (answer)
 D) Pulmonary hypertension

38
Q
  1. A 56-year-old man comes to the emergency department with symptoms of heart failure. Which of the following treatments is appropriate?

    A) Oral aspirin


B) Intravenous furosemide

C) Intravenous nitroglycerin 
 D) Beta-blockers

4. A 63-year-old man presents with exertional dyspnea and a heart murmur. Echocardiography shows an aortic valve area of 0.8 cm2. What is the next best step?
 A) Medical management with beta-blockers

B) Repeat echocardiography in 6 months

C) Aortic valve replacement after coronary angiogram
D) Percutaneous balloon valvuloplasty


A
  1. A 56-year-old man comes to the emergency department with symptoms of heart failure. Which of the following treatments is appropriate?

    A) Oral aspirin


B) Intravenous furosemide

C) Intravenous nitroglycerin (answer)
 D) Beta-blockers

4. A 63-year-old man presents with exertional dyspnea and a heart murmur. Echocardiography shows an aortic valve area of 0.8 cm2. What is the next best step?
 A) Medical management with beta-blockers

B) Repeat echocardiography in 6 months

C) Aortic valve replacement after coronary angiogram (answer)

D) Percutaneous balloon valvuloplasty


39
Q
  1. Which of the following statements about angina pectoris is true?

    A) Pain is usually localized to the left arm.

    B) It is not relieved by nitroglycerin.

    C) Substernal, characterized by burning, heavy, or squeezing pain relieved by rest or nitroglycerin. 

    D) It does not occur with emotional stress.

  2. Which drugs can cause lower extremity peripheral edema?
 A) Beta-blockers

    B) Diuretics

    C) Calcium channel blockers 
 D) ACE inhibitors

A
  1. Which of the following statements about angina pectoris is true?

    A) Pain is usually localized to the left arm.

    B) It is not relieved by nitroglycerin.

    C) Substernal, characterized by burning, heavy, or squeezing pain relieved by rest or nitroglycerin. (answer)

    D) It does not occur with emotional stress.

  2. Which drugs can cause lower extremity peripheral edema?
 A) Beta-blockers

    B) Diuretics

    C) Calcium channel blockers (answer)
 D) ACE inhibitors

40
Q
  1. Which of the following best describes the sound of mitral regurgitation?

    A) High-pitched diastolic murmur radiating to the carotids

    B) Low-pitched diastolic murmur heard at the apex

    C) Systolic ejection murmur heard at the apex and radiating to the axilla 
 D) Continuous machinery murmur

  2. Which of the following statements about syncope is correct?

    A) It is always associated with cardiac arrhythmias.

    B) Neurally mediated syncope includes only vasovagal syncope.

    C) Neurally mediated syncope includes vasovagal, situational syncope, and carotid sinus syndrome.
    D) It is never associated with postural hypotension.

A
  1. Which of the following best describes the sound of mitral regurgitation?

    A) High-pitched diastolic murmur radiating to the carotids

    B) Low-pitched diastolic murmur heard at the apex

    C) Systolic ejection murmur heard at the apex and radiating to the axilla (answer)
 D) Continuous machinery murmur

  2. Which of the following statements about syncope is correct?

    A) It is always associated with cardiac arrhythmias.

    B) Neurally mediated syncope includes only vasovagal syncope.

    C) Neurally mediated syncope includes vasovagal, situational syncope, and carotid sinus syndrome. (answer)

    D) It is never associated with postural hypotension.